limit of implicit sequences

The name of the pictureThe name of the pictureThe name of the pictureClash Royale CLAN TAG#URR8PPP












3














In a HS problem, we study the functions



$$f_n:x longmapsto (x-n)ln x - xln(x-n)$$ for $n$ natural with $nge 5$



First questions just ask the domain, the monotonicity ($f_n$ is defined for $x> n$ and is strictly decreasing on its domain) then by IVT we show that the equation $f_n(x)=0$ has a unique solution $alpha_n$ that satisifes $n+1 < alpha_n <n+2$.



From last inequality it's clear that $(alpha_n)$ diverges to $+infty$.



The last question asks to prove that $limlimits_n to +infty (alpha_n - n)=1$ and then to evaluate $limlimits_n to +infty (alpha_n+1 - alpha_n)$.



I'm stuck with this last question.










share|cite|improve this question




























    3














    In a HS problem, we study the functions



    $$f_n:x longmapsto (x-n)ln x - xln(x-n)$$ for $n$ natural with $nge 5$



    First questions just ask the domain, the monotonicity ($f_n$ is defined for $x> n$ and is strictly decreasing on its domain) then by IVT we show that the equation $f_n(x)=0$ has a unique solution $alpha_n$ that satisifes $n+1 < alpha_n <n+2$.



    From last inequality it's clear that $(alpha_n)$ diverges to $+infty$.



    The last question asks to prove that $limlimits_n to +infty (alpha_n - n)=1$ and then to evaluate $limlimits_n to +infty (alpha_n+1 - alpha_n)$.



    I'm stuck with this last question.










    share|cite|improve this question


























      3












      3








      3







      In a HS problem, we study the functions



      $$f_n:x longmapsto (x-n)ln x - xln(x-n)$$ for $n$ natural with $nge 5$



      First questions just ask the domain, the monotonicity ($f_n$ is defined for $x> n$ and is strictly decreasing on its domain) then by IVT we show that the equation $f_n(x)=0$ has a unique solution $alpha_n$ that satisifes $n+1 < alpha_n <n+2$.



      From last inequality it's clear that $(alpha_n)$ diverges to $+infty$.



      The last question asks to prove that $limlimits_n to +infty (alpha_n - n)=1$ and then to evaluate $limlimits_n to +infty (alpha_n+1 - alpha_n)$.



      I'm stuck with this last question.










      share|cite|improve this question















      In a HS problem, we study the functions



      $$f_n:x longmapsto (x-n)ln x - xln(x-n)$$ for $n$ natural with $nge 5$



      First questions just ask the domain, the monotonicity ($f_n$ is defined for $x> n$ and is strictly decreasing on its domain) then by IVT we show that the equation $f_n(x)=0$ has a unique solution $alpha_n$ that satisifes $n+1 < alpha_n <n+2$.



      From last inequality it's clear that $(alpha_n)$ diverges to $+infty$.



      The last question asks to prove that $limlimits_n to +infty (alpha_n - n)=1$ and then to evaluate $limlimits_n to +infty (alpha_n+1 - alpha_n)$.



      I'm stuck with this last question.







      calculus sequences-and-series






      share|cite|improve this question















      share|cite|improve this question













      share|cite|improve this question




      share|cite|improve this question








      edited Dec 27 '18 at 10:27







      Oussama Sarih

















      asked Dec 27 '18 at 8:49









      Oussama SarihOussama Sarih

      47827




      47827




















          2 Answers
          2






          active

          oldest

          votes


















          3














          For the first part note that



          $$fracalpha_n-nlog(alpha_n - n) = fracalpha_nlog(alpha_n) $$



          Since $alpha_n to infty$, the right hand side diverges to infinity so the left hand side must diverge to infinity as well. Since $1 < alpha_n - n < 2$ for all $n$, this implies that $log(alpha_n-n) to 0$ i.e. $alpha_n - n to 1$.



          For the second part



          $$ beginalign lim(alpha_n+1 - alpha_n) & = lim((a_n+1 - (n+1)) - (alpha_n - n) + 1) \ & = lim(alpha_n+1 - (n+1)) - lim(alpha_n - n) + 1 \ & = 1 - 1 + 1 \ & = 1 endalign$$






          share|cite|improve this answer




























            3














            This could be a stupid answer.



            You look for the zero of
            $$f_n=(x-n)log( x) - xlog(x-n)$$ and you showed that the root is between $n+1$ and $n+2$. So,let $x=n+1+epsilon$ to make
            $$f_n=(1+epsilon ) log (n+1+epsilon)-(n+1+epsilon) log (1+epsilon)$$ Expand it as a Taylor series for infinitely large values of $n$ to get
            $$f_n=-n log (1+epsilon)-(1+epsilon)log left(frac1+epsilon nright)+Oleft(frac1nright)$$ So, if $nto infty$, $epsilonto 0$






            share|cite|improve this answer




















              Your Answer





              StackExchange.ifUsing("editor", function ()
              return StackExchange.using("mathjaxEditing", function ()
              StackExchange.MarkdownEditor.creationCallbacks.add(function (editor, postfix)
              StackExchange.mathjaxEditing.prepareWmdForMathJax(editor, postfix, [["$", "$"], ["\\(","\\)"]]);
              );
              );
              , "mathjax-editing");

              StackExchange.ready(function()
              var channelOptions =
              tags: "".split(" "),
              id: "69"
              ;
              initTagRenderer("".split(" "), "".split(" "), channelOptions);

              StackExchange.using("externalEditor", function()
              // Have to fire editor after snippets, if snippets enabled
              if (StackExchange.settings.snippets.snippetsEnabled)
              StackExchange.using("snippets", function()
              createEditor();
              );

              else
              createEditor();

              );

              function createEditor()
              StackExchange.prepareEditor(
              heartbeatType: 'answer',
              autoActivateHeartbeat: false,
              convertImagesToLinks: true,
              noModals: true,
              showLowRepImageUploadWarning: true,
              reputationToPostImages: 10,
              bindNavPrevention: true,
              postfix: "",
              imageUploader:
              brandingHtml: "Powered by u003ca class="icon-imgur-white" href="https://imgur.com/"u003eu003c/au003e",
              contentPolicyHtml: "User contributions licensed under u003ca href="https://creativecommons.org/licenses/by-sa/3.0/"u003ecc by-sa 3.0 with attribution requiredu003c/au003e u003ca href="https://stackoverflow.com/legal/content-policy"u003e(content policy)u003c/au003e",
              allowUrls: true
              ,
              noCode: true, onDemand: true,
              discardSelector: ".discard-answer"
              ,immediatelyShowMarkdownHelp:true
              );



              );













              draft saved

              draft discarded


















              StackExchange.ready(
              function ()
              StackExchange.openid.initPostLogin('.new-post-login', 'https%3a%2f%2fmath.stackexchange.com%2fquestions%2f3053720%2flimit-of-implicit-sequences%23new-answer', 'question_page');

              );

              Post as a guest















              Required, but never shown

























              2 Answers
              2






              active

              oldest

              votes








              2 Answers
              2






              active

              oldest

              votes









              active

              oldest

              votes






              active

              oldest

              votes









              3














              For the first part note that



              $$fracalpha_n-nlog(alpha_n - n) = fracalpha_nlog(alpha_n) $$



              Since $alpha_n to infty$, the right hand side diverges to infinity so the left hand side must diverge to infinity as well. Since $1 < alpha_n - n < 2$ for all $n$, this implies that $log(alpha_n-n) to 0$ i.e. $alpha_n - n to 1$.



              For the second part



              $$ beginalign lim(alpha_n+1 - alpha_n) & = lim((a_n+1 - (n+1)) - (alpha_n - n) + 1) \ & = lim(alpha_n+1 - (n+1)) - lim(alpha_n - n) + 1 \ & = 1 - 1 + 1 \ & = 1 endalign$$






              share|cite|improve this answer

























                3














                For the first part note that



                $$fracalpha_n-nlog(alpha_n - n) = fracalpha_nlog(alpha_n) $$



                Since $alpha_n to infty$, the right hand side diverges to infinity so the left hand side must diverge to infinity as well. Since $1 < alpha_n - n < 2$ for all $n$, this implies that $log(alpha_n-n) to 0$ i.e. $alpha_n - n to 1$.



                For the second part



                $$ beginalign lim(alpha_n+1 - alpha_n) & = lim((a_n+1 - (n+1)) - (alpha_n - n) + 1) \ & = lim(alpha_n+1 - (n+1)) - lim(alpha_n - n) + 1 \ & = 1 - 1 + 1 \ & = 1 endalign$$






                share|cite|improve this answer























                  3












                  3








                  3






                  For the first part note that



                  $$fracalpha_n-nlog(alpha_n - n) = fracalpha_nlog(alpha_n) $$



                  Since $alpha_n to infty$, the right hand side diverges to infinity so the left hand side must diverge to infinity as well. Since $1 < alpha_n - n < 2$ for all $n$, this implies that $log(alpha_n-n) to 0$ i.e. $alpha_n - n to 1$.



                  For the second part



                  $$ beginalign lim(alpha_n+1 - alpha_n) & = lim((a_n+1 - (n+1)) - (alpha_n - n) + 1) \ & = lim(alpha_n+1 - (n+1)) - lim(alpha_n - n) + 1 \ & = 1 - 1 + 1 \ & = 1 endalign$$






                  share|cite|improve this answer












                  For the first part note that



                  $$fracalpha_n-nlog(alpha_n - n) = fracalpha_nlog(alpha_n) $$



                  Since $alpha_n to infty$, the right hand side diverges to infinity so the left hand side must diverge to infinity as well. Since $1 < alpha_n - n < 2$ for all $n$, this implies that $log(alpha_n-n) to 0$ i.e. $alpha_n - n to 1$.



                  For the second part



                  $$ beginalign lim(alpha_n+1 - alpha_n) & = lim((a_n+1 - (n+1)) - (alpha_n - n) + 1) \ & = lim(alpha_n+1 - (n+1)) - lim(alpha_n - n) + 1 \ & = 1 - 1 + 1 \ & = 1 endalign$$







                  share|cite|improve this answer












                  share|cite|improve this answer



                  share|cite|improve this answer










                  answered Dec 27 '18 at 9:09









                  ODFODF

                  1,381510




                  1,381510





















                      3














                      This could be a stupid answer.



                      You look for the zero of
                      $$f_n=(x-n)log( x) - xlog(x-n)$$ and you showed that the root is between $n+1$ and $n+2$. So,let $x=n+1+epsilon$ to make
                      $$f_n=(1+epsilon ) log (n+1+epsilon)-(n+1+epsilon) log (1+epsilon)$$ Expand it as a Taylor series for infinitely large values of $n$ to get
                      $$f_n=-n log (1+epsilon)-(1+epsilon)log left(frac1+epsilon nright)+Oleft(frac1nright)$$ So, if $nto infty$, $epsilonto 0$






                      share|cite|improve this answer

























                        3














                        This could be a stupid answer.



                        You look for the zero of
                        $$f_n=(x-n)log( x) - xlog(x-n)$$ and you showed that the root is between $n+1$ and $n+2$. So,let $x=n+1+epsilon$ to make
                        $$f_n=(1+epsilon ) log (n+1+epsilon)-(n+1+epsilon) log (1+epsilon)$$ Expand it as a Taylor series for infinitely large values of $n$ to get
                        $$f_n=-n log (1+epsilon)-(1+epsilon)log left(frac1+epsilon nright)+Oleft(frac1nright)$$ So, if $nto infty$, $epsilonto 0$






                        share|cite|improve this answer























                          3












                          3








                          3






                          This could be a stupid answer.



                          You look for the zero of
                          $$f_n=(x-n)log( x) - xlog(x-n)$$ and you showed that the root is between $n+1$ and $n+2$. So,let $x=n+1+epsilon$ to make
                          $$f_n=(1+epsilon ) log (n+1+epsilon)-(n+1+epsilon) log (1+epsilon)$$ Expand it as a Taylor series for infinitely large values of $n$ to get
                          $$f_n=-n log (1+epsilon)-(1+epsilon)log left(frac1+epsilon nright)+Oleft(frac1nright)$$ So, if $nto infty$, $epsilonto 0$






                          share|cite|improve this answer












                          This could be a stupid answer.



                          You look for the zero of
                          $$f_n=(x-n)log( x) - xlog(x-n)$$ and you showed that the root is between $n+1$ and $n+2$. So,let $x=n+1+epsilon$ to make
                          $$f_n=(1+epsilon ) log (n+1+epsilon)-(n+1+epsilon) log (1+epsilon)$$ Expand it as a Taylor series for infinitely large values of $n$ to get
                          $$f_n=-n log (1+epsilon)-(1+epsilon)log left(frac1+epsilon nright)+Oleft(frac1nright)$$ So, if $nto infty$, $epsilonto 0$







                          share|cite|improve this answer












                          share|cite|improve this answer



                          share|cite|improve this answer










                          answered Dec 27 '18 at 9:32









                          Claude LeiboviciClaude Leibovici

                          119k1157132




                          119k1157132



























                              draft saved

                              draft discarded
















































                              Thanks for contributing an answer to Mathematics Stack Exchange!


                              • Please be sure to answer the question. Provide details and share your research!

                              But avoid


                              • Asking for help, clarification, or responding to other answers.

                              • Making statements based on opinion; back them up with references or personal experience.

                              Use MathJax to format equations. MathJax reference.


                              To learn more, see our tips on writing great answers.





                              Some of your past answers have not been well-received, and you're in danger of being blocked from answering.


                              Please pay close attention to the following guidance:


                              • Please be sure to answer the question. Provide details and share your research!

                              But avoid


                              • Asking for help, clarification, or responding to other answers.

                              • Making statements based on opinion; back them up with references or personal experience.

                              To learn more, see our tips on writing great answers.




                              draft saved


                              draft discarded














                              StackExchange.ready(
                              function ()
                              StackExchange.openid.initPostLogin('.new-post-login', 'https%3a%2f%2fmath.stackexchange.com%2fquestions%2f3053720%2flimit-of-implicit-sequences%23new-answer', 'question_page');

                              );

                              Post as a guest















                              Required, but never shown





















































                              Required, but never shown














                              Required, but never shown












                              Required, but never shown







                              Required, but never shown

































                              Required, but never shown














                              Required, but never shown












                              Required, but never shown







                              Required, but never shown






                              Popular posts from this blog

                              How to check contact read email or not when send email to Individual?

                              Bahrain

                              Postfix configuration issue with fips on centos 7; mailgun relay